Which one of the following must be true?

Anevay on February 9, 2021

A v. D

I was able to narrow it down to A and D based on the previous hypothetical. How do I properly deduce the correct answer?

Reply
Create a free account to read and take part in forum discussions.

Already have an account? log in

Victoria on February 9, 2021

Hi @Anevay,

Happy to help!

Rule 3 tells us that G demonstrates neither H nor M.

Rule 4 tells us that L demonstrates neither H nor T.

Since each of the tasks must be demonstrated and there are only three volunteers, this means that F must demonstrate H.

This is directly restated by answer choice (A), making it our correct answer.

Answer choice (D) does not have to be true. As an example, it is entirely possible that G demonstrates S and T.

V: L F G L F G
D: P W T M H S
1 2 3 4 5 6

Notice that this meets all of our conditions:

1) F demonstrates W before G demonstrates either T or S.
2) F performs neither the first nor the last demonstration; F performs the second and the fifth demonstration.
3) G demonstrates neither H nor M; G demonstrates T and S.
4) L demonstrates neither H nor T; L demonstrates P and M.
5) M is demonstrated fourth, following T which is demonstrated third.

Hope this helps! Please let us know if you have any further questions.